« first day (38 days earlier)      last day (536 days later) » 
00:00 - 15:0015:00 - 00:00

12:06 AM
0
Q: Finding a vector field

SademGive a formula $F=M(x,y)i + N(x,y)j$ for the vector field in the plane that has the property that F points toward the origin with magnitude inversely proportional to the square of the distance from (x,y) to the origin. (The field is not defined at (0,0)). I first found the norm of the vector $|F...

 
12:30 AM
0
Q: Teaching integration to kids

Madelyn ShroederI have been selected by my college to teach integration to kids in the age group of 8-12. I am an engineering major who has finished Calculus 1 and 2 but I have no idea how to teach integration from scratch to kids that small and at the same time make it fun for them. I am asked to create a lesso...

0
Q: Showing that points are in the Mandelbrot set

GuestgreatI am given ( a simplistic definition I think ) of Mandelbrot set: M- set of complex numbers $c \in \mathbb{C}$ s.t. the sequence $(z_n)$ is bounded where $z_0=0 , z_{n+1}=z_n^2+c $ Need to show that: (i)$-2\in M$ (ii) $1/4 \in M$. Am I missing something here or is it kind of trivial? (i) ...

0
Q: Scheduling problem on graph (G, U, V).

user43498Consider a bipartite graph $(G, U, V)$. Each $v$ in $V$ represents a soccer team, and each $u$ in $U$ represents a mini-tournament needs to be scheduled. If $u_i$ and $u_j$ share no common neighbor, these two tournament can be scheduled on the same day. Similarly, one can schedule multiple tourn...

Welcome to Math.SE, Guestgreat. Consider adding a tag for a broader subject area to which the question belongs. Some of these tags might fit. (autocomment)Normal Human 28 secs ago
 
0
Q: Un-needingly losing reputation?

David ColeWhy do we lose reputation when we downvote an answer? I downvoted an answer the other day that was advertising an acne product, and lost 1 reputation. Like what? There's no reason to lose reputation for doing something productive.

 
0
Q: Are any 2 groups of order 6 isomorphic?

Théodore RozencwajgSo my question is pretty simple : Are any groups of order 6 isomorphic ? I would say no, but I know that if the groups are cyclic then yes. If the answer is indeed no, could I please have a counter-example ? Thank you.

Question contains please. Are any 2 groups of order 6 isomorphic?
0
Q: To find the points on the surface

Vinayvicky1107Find a point on the surface 1/x+1/y+1/z=1 (x>0,y>0,z>0) such as the tangent plane at this point is parallel to the plane x+2y+3z=0

0
Q: Can anyone explain relation between <> and ⌊⌋ symbols (in probability theory?)?

PtruI see this: $<nx> = nx - ⌊xn⌋$ in one of my exercise solutions for probability theory class. I found on wikipedia that <> means average and ⌊⌋ means greatest integer with the following examples: $⌊4⌋ = 4, ⌊2.1⌋ = 2, ⌊2.9⌋ = 2, ⌊−2.6⌋ = −3$ Now I'm wondering why $⌊2.9⌋ = 2$ and $⌊−2.6⌋ = −3$. ...

0
Q: Definite integral of $\int_{-\pi}^{\pi} {x^{1000}\sin x}dx$

user989894$\int_{-\pi}^{\pi} {x^{1000}\sin x}dx$ As I understand, the definite integral equals to $0$. However, what are the ways to show that properly?

0
Q: question about epsilon delta proof

PiFarmer86I just had a few questions concerning the epsilon delta proof of limits. To be more precise, I always get lost at the part where we take $\delta$ to be the minimum of two real numbers. Let me provide a concrete example to work with. Consider $\displaystyle\lim_{x \to 2}x^2 = 4$ Given $\epsilon >...

Words such as question are uninformative in titles. Please edit the title so that it better describes the specifics of your question. Do not hesitate to make it longer or include a formula if needed. More tips here. (autocomment)Normal Human 21 secs ago
0
Q: The integers $x+k(\frac{n}{gcd(a,n)}$ with $k=0,...,gcd(a,n)-1$ are the only solutions to the equation $ax=b mod n$

user278055Here, x is the unique solution mod $\frac{n}{gcd(a,n)}$ Let $gcd(a,n)>1$ and consider the equation: $\frac{a}{gcd(a,n)}x = \frac{b}{gcd(a,n)} mod \frac{n}{gcd(a,n)}$

0
Q: Zero homomorphisms and fields

Théodore RozencwajgMy question is the following : Let n ≥ 1 be an integer. Every homomorphism of groups $f$: ℤ/nℤ → ℤ is the zero homomorphism. a) Yes. b) No. c) Depends on n. I don't really know how to go with this one. Could someone re-explain briefly the zero homomorphism ? Thank you.

0
Q: Help on solving the equation $\frac{\sqrt{a+x}}{\sqrt{a}+\sqrt{a+x}}=\frac{\sqrt{a-x}}{\sqrt{a}-\sqrt{a-x}$

nullgeppettoCould you give me some help on finding the roots (if any) of the following equation: $$ \frac{\sqrt{a+x}}{\sqrt{a}+\sqrt{a+x}}=\frac{\sqrt{a-x}}{\sqrt{a}-\sqrt{a-x}} $$ I tried to apply some classic approaches, but I had no luck... Could you lend me a hand? Thanks in advance!

0
Q: Rate of Change of a Sum

Max SDisclaimer: Probably stupid question Is there a polynomial representation of the rate of change with respect to $n$ of $$f(n)=2n-3+\sum_{m=0}^{n-3}{n-3-m}$$ and if so, how can I find it? (For those wondering, this is the maximum number of undirected connections between $n$ vertices.)

0
Q: What is linear approximation?

Mone Skratt HenryWhat is the purpose of linear approximation questions? For example, one question reads: (a) Use the Linear Approximation for f(x) = ln(x) at a = 1 to estimate ln(0.84)... (b) That Linear Approximation has error... How would I complete this question?

0
Q: Let $L=\int^1_0 \frac{dx}{1+x^8}$. Then

rajendraLet $L=\int^1_0 \frac{dx}{1+x^8}$. Then A. L<1 B. L>1 c. L D. L>$\frac{\pi}{4}$

0
Q: Showing that a recursive sequence is bounded

LostUserI need to show that the sequence $x_n$ defined as : $x_0=0 , x_{n+1}=x_n^2+1/4 $ is bounded. Not sure how to approach this, some hints would be greatly appreciated :)

0
Q: Relations on groups.

Théodore RozencwajgLet G be a group and H a subgroup of G. Define a relation on elements of G by saying that a ~ b if b$^-$¹ a ∈ H. This relation is : a) reflexive and symmetric, but transitive only if G is abelian. b) relfexive and transitive, but symmetric only if G is abelian. c) reflexive, symmetric and trans...

Short title. Relations on groups.
0
Q: How to prove the left pseudo-inverse has a specific form?

FalimondEquipped only with the knowledge that $MA = I$, $A$ is one-to-one and the output weighting matrix is $W$, how shall we prove that $(A^HWA)^{-1}A^HW = M$ is the form for all left pseudo-inverses of A? My thinking is that first we can show that $M = (A^HWA)^{-1}A^HW$ is at least one of the (possib...

Consider adding a tag for a broader subject area to which the question belongs. Some of these tags might fit. (from a bot)Normal Human 21 secs ago
0
Q: How do you factor $x^2-x-1$?

Jacob WheelerI know you can't have all integers, but how do you factor this anyway? Wolfram|Alpha gives me $-\frac{1}{4} (1+\sqrt{5}-2 x) (-1+\sqrt{5}+2 x)$. Cymath gives me $(x-\frac{1+\sqrt{5}}{2})(x-\frac{1-\sqrt{5}}{2})$. The closest I can get is $(x+1)(x-1)-x$. So how do I get a nice answer like the o...

0
Q: How do we study the limit $\lim_{n\to\infty}\frac{n(n-1)...(n-k_n+1}{n^{k_n}}$ wrt the relationship of $n$ and $k_n$

HarperI think that when $k_n$ is big enough the limit will be 0, and when $k_n$ is small enough the limit will be going to 1. How do we make a formal analysis?

0
Q: Solve the recurrence relation : $f(n) = 1 + \dfrac{f(n) + f(n-1) + \cdots + f(1)}{n+1}$

pchakrabFor naturals $n$, $f(n) = 1 + \dfrac{f(n) + f(n-1) + \cdots + f(1)}{n+1}$. What is $f(n)$? This is not a homework problem. Is there a general method to solve these recurrence relations? I will appreciate if someone directs me to a short tutorial/book to learn about solving recurrence relations.

0
Q: Show $f(x)=\sqrt{x}$ is uniformly continuous on $[0,\infty)$

Simple Show $f(x)=\sqrt{x}$ is uniformly continuous on $[0,\infty)$ Let $\epsilon>0$, then there exists a $\delta=\epsilon^2$ such that $|x_1-x_2|<\delta$ for all $x_1,x_2\in[0,\infty)$. $\begin{align}|f(x_1)-f(x_2)|&=|\sqrt{x_1}-\sqrt{x_2}|\\&<\sqrt{|x_1-x_2|}\\&<\sqrt{\delta}\\&=\epsilon\end{a...

Tall formulas in titles break the layout of question lists. Please replace \dfrac with \frac in the title. (autocomment)Normal Human 21 secs ago
0
Q: Transitive actions on sets.

Théodore RozencwajgDoes there exist a transitive action of S4 on the set {1,2,3,4,5} ? I would say no, because the cardinality of our set is bigger than 4, but I am not sure how to prove this. We can’t obtain the element {5} with the cycles of S4 , that’s the way I would go for it. Thank you.

0
Q: Proving that the limit as x tends to infinity of (e^x)/(e^x + x) is equal to 1

HenrikI’m generally pretty good at proving limits using epsilon-delta, but on this one I’m stuck — and have been for days. This is the problem I’m talking about: \begin{equation} \lim_{x\to\infty}\frac{e^x}{e^x + x} = 1. \end{equation} I know that I need to let $\epsilon>0$ and choose an $N$ suc...

0
Q: Ito integral for brownian motion

DreamerI know that because $W_t$ is a martingale, $$\int_{0}^{T} W_t dW_t = 0$$ then what should the value for this equation be: $$\int_{0}^{T} W_t^{n}dW_t$$

0
Q: Can somebody expain to me Pascal's triangle? Just tell me about it. Patterns, numbers, etc.

JoeAre there any patterns, symbols, etc. in Pascal's triangle? For example, how do you draw a Pascal's triangle?

0
Q: Computing the integral

IdiotfromPrincetonI need help to compute the following indefinite integral: $$\int_{-\infty}^{\infty}\frac{z^4}{1+z^8}dz$$ I need to use Cauchy's residue theorem. I can write that $z^8+1=z^8-i^2=(z^4-i)(z^4+1)$. How do I proceed? Please give a methodological answer so that I can solve other questions too.

Short title. Question contains please. Computing the integral
0
Q: Find all x-values where the tangent line is horizontal:

NickFind all x-values where the tangent line is horizontal: y=(x+1)^4(x-2)^3

0
Q: Conway's Soldiers

RachelI've been working on a modification to the standard Conway's soldiers game. In Conway's soldiers, we have an endless number of soldiers in a grid of squares at and below point 0 North, and I can move soldiers left, right, up, and down by having them jump over other soldiers, which makes those ...

Short title. Conway's Soldiers
0
Q: Proof by induction: Number of subsets of cardinality 2

nullgeppettoWe would like to prove by induction that the number of the subsets of cardinality $2$ of a finite set with $n$ elements is given by $\frac{n(n-1)}{2}$. I know the reason why this is true, but how could I use it in the induction process? Something seems to be missing here...

0
Q: Units of a dual numbers ring.

Théodore RozencwajgWhat are the units of ℂ[ε] ? ℂ-{0} would be the units of ℂ, I get 3 other possibilities : a) {a+bε : a,b ∈ ℂ, a not equal to 0} b) {1} c) {bε : b ∈ ℂ, b not equal to 0} I would go for a) but not sure really. Thank you.

 
2:21 AM
1
Q: What is special about the `undefined` tag?

Eric PlatonFor some reason I encountered the undefined tag for the first time earlier today, and I am not sure why the community seems to keep it, whereas an old tag like option was eradicated for irrelevancy (agreeing with that conclusion). The undefined tag is usually not alone, as it can apply to almost...

 
0
Q: Need help understanding concept about irrediducible polynomials in finite fields

QualityHello I am really needing for someone to help for me to understand the following; I know that in fields, quadratics and cubics are irreducible if and only if they have roots. My question is, what is the form of these roots. For example to explain my confusion, consider the field with 121 elemen...

0
Q: Elimination of quantifiers

Mary StarWhat does it mean that a theory admits constructive elimination of quantifiers? A theory admits elimination of quantifiers when each formula of the theory is equivalent to a quanifier-free formula, right? But what is meant when we use the term "constructive" ?

0
Q: Regular Pumping Lemma

Marquackenter image description here Determine if each language is regular or not-regular. The former justi ed by providing a minimal DFA which accepts the said language and the latter by using the Pumping Lemma for Regular Expressions. Please help. Thanks :)

Short title. Question contains please. Regular Pumping Lemma
0
Q: Double limit and iterated limit on $\epsilon - \delta$ proof

user89627Let $f(x,y)$ be a real-valued function defined on an open set $S$ containing the origin. Prove the following by $\epsilon - \delta$ definition: If there exists: $$\lim_{(x,y)\to (0,0)} f(x,y)=L$$, and there exists: $$\lim_{x \to 0}\lim_{y \to 0} f(x,y)=L_{12}$$, then $L=L_{12}$ I'm trying to wo...

0
Q: Likelihood function - Gamma Distribution

ano_aCan you help me find the likelihood function for the below? $$ X_i \sim \pi_0Gamma(1, β_1) + π_1Gamma(1, β_2) $$

0
Q: Principal ideals in rings.

Théodore RozencwajgConsider the following list of principal ideals (2), (3), (5), (6) in the ring ℤ/14ℤ. There are : a) Only one ideal in this list. b) Two distinct ideals in this list. c) Three distinct ideals in this list. d) Four distinct ideals in this list. I know the answer is related to divisors of 14, b...

0
Q: Determining limit using squeeze theorem.

bebecaHow do I apply the squeeze theorem for finding the limit of this problem? Sorry, not sure how to format on here. lim(|x|cos^2(1/x)) x->0

0
Q: Singleton Tuple Equality

IMac McGrawI gather that mathematicians distinguish between sets of a single element and the element itself, that is $\forall x,\ x \ne \{x\}$. However, in regard to tuples, is it true that $\forall x,\ x = (x)$? $*$ Where $(x)$ is the singleton tuple with element $x$.

0
Q: Please help checking with central limit

PersonaACan someone please help me to check over , I get a different answer then is written as the solution Suppose we are told that the weight of each gum ball ( in centigram) is given by the gamma distribution function, with $α=25$ and $β=2$ . We are wanting to know the probability that 100 gum bal...

Words such as please, help are uninformative in titles. Please edit the title so that it better describes the specifics of your question. Do not hesitate to make it longer or include a formula if needed. More tips here. (autocomment)Normal Human 21 secs ago
0
Q: Find the derivative of the function using the limit definition of derivative:

NickFind the derivative of the function using the limit definition of derivative: f(x)=square root(x)

0
Q: Algebra question with ln

user297486I was helping out my little brother with his math homework and I came across a question I'm pretty sure is unsolvable without the use of a graphing calculator. It's been a while since I've done math, so can anyone confirm for me? $\frac{120000}{1+48e^{-.015t} } = 24e^{.055t}$

Welcome to Math.SE, user297486. Words such as question do not add information to titles. Please edit the title so that it better describes the specifics of your question. Do not hesitate to make it longer or include a formula if needed. More tips here. (autocomment)Normal Human 21 secs ago
0
Q: Local Solid Angle Units

user173897This is a cultural question: Are there any, even moderately or historically used, units that measure solid angles which are not steradians? Basically, is there a unit x such that x:sr::grad:rad?

Consider adding a tag for a broader subject area to which the question belongs. Some of these tags might fit. (autocomment)Normal Human 21 secs ago
0
Q: taylor series with error at most 10^-3

Mikefloat cosine(float x, int j) { float val = 1; for (int k = j - 1; k >= 0; --k) val = 1 - x*x/(2*k+2)/(2*k+1)*val; return val; } int main( void ) { for( double x = 0; x <= PI/4; x += 0.9999 ) { if(cosine(x, 2) <= 0.001) { printf("cos(x) : %10g ...

Questions tend to get more attention when they have a tag for a broad area of mathematics relevant to the question. Some of these tags might fit. (from a bot)Normal Human 21 secs ago
0
Q: Units for Measuring Rotation of a 3-Sphere in Four Dimensions

user173897In four dimensions, there can be two orthogonal axes of simultaneous rotation (of two planes), right? Does that mean that we can measure such rotations of a 3-sphere in solid angles, much as we measure the rotation of a circle (or 2-sphere- still one axis) in radians? Would there be a notion of "...

Questions tend to get more attention when they have a tag for a broad area of mathematics relevant to the question. Some of these tags might fit. (from a bot)Normal Human 21 secs ago
0
Q: how to draw phase portrait like THIS?(not StreamPlot or VectorPlot)

minori11I really try hard to draw phase portrait shown below by mathematica or matlab, but I failed. Please help enter image description here

Short question. Question contains please. Tagged mathematica. how to draw phase portrait like THIS?(not StreamPlot or VectorPlot)
0
Q: Potentially A Pigeon-Hole?

LiamI'm working on a proof where I want any subset of $n+1$ distinct integers chosen from $\{1,2,...,2n\}$ has at least two numbers such that one divides the other. I have a feeling that this may be a problem related to modular arithmetic on the set $[2n]$, but I am having issues figuring out the equ...

0
Q: name or formal description for the shape of a Pac-Man board

rich remerI'm looking for a name for the (finite, boundless, impossible) shape created by taking a square and wrapping it so that the opposite edges are coterminous (I think that's the correct usage, it's been a while). Effectively, this is the shape formed by the Pac-Man board, where you exit the right s...

Welcome to Math.SE, rich remer. Consider adding a tag for a broader subject area to which the question belongs. Some of these tags might fit. (from a bot)Normal Human 21 secs ago
0
Q: Express the following complex numbers in standard form

bella((root3/2)+(i/2))^25 I know that you have to put it in the form costheta+isintheta but i'm not sure how to go about it.

This site uses MathJax formatting of formulas. More tips here. (autocomment)Normal Human 21 secs ago
0
Q: basic question on limits and sequences

lkrI'm wondering if I have a sufficient proof of the following: If $(a_n)$ is a sequence such that $\lim_{n \rightarrow \infty}a_n=A$, then $\lim_{n \rightarrow \infty}\frac{a_1+...+a_n}{n}=A$. My approach: For all $\epsilon > 0$, there exists $N$ such that for all $k>N$, $|a_k -A|<\epsilon$. S...

Words such as question do not add information to titles. Please edit the title so that it better describes the specifics of your question. Do not hesitate to make it longer or include a formula if needed. More tips here. (from a bot)Normal Human 21 secs ago
 
3:47 AM
0
Q: Integral of function with absolute variable

user3436815 integrate and how absolute variable affect on the final result if Limit -0.5 to 0.5.

This site uses MathJax formatting of formulas. More tips here. (from a bot)Normal Human 21 secs ago
0
Q: solve the following equations for x exists in c

bellaa. z^3 = (1-iroot3)8 b. z^2 - (3-2i)z + (1-3i) = 0 c. z^4 + 1 + iroot3 = 0 I know for the last two you start off using the quad form but i'm not sure what to mainly do for any of them.

This site uses MathJax formatting of formulas. More tips here. (from a bot)Normal Human 21 secs ago
0
Q: A question about convolution using a graphical approach

adesmond01How do you convolve multiple dirac-delta functions a rect function? Is the below convolution correct? Thank you! Anthonya picture of graphical convolution

Words such as question are uninformative in titles. Please edit the title so that it better describes the specifics of your question. Do not hesitate to make it longer or include a formula if needed. More tips here. (from a bot)Normal Human 21 secs ago
0
Q: Finding an inverse for a linear combination of congruence classes

Jch2016Given: x^2 - 2 being irreducible in Q[x], 2x+5 and x^2 -2 relatively prime in Q[x]. Congruence class [2x+5] is a unit in the ring Q[x]/(x^2 - 2). Find the inverse u(x) given (2x+5)(u(x)) + (x^2-2)(v(x)) = 1.

Welcome to Math.SE, Jch2016. This site uses MathJax formatting of formulas. More tips here. (from a bot)Normal Human 21 secs ago
0
Q: Prove from first principles that the derivative of two functions f(x), and g(x) is the sum of the derivatives

jackI do not know how to approach this question. I have never dealt with first principle questions which involved a g(x) so I am a bit confused how I would go about it in the first place. Please help me!

0
Q: Logarithmic differentiation trouble with bottom of fraction

windy401$y=\frac{(2x+3)^9}{\sqrt x(x^2-x)^6}$ I switched it to $ln(y)=9ln(2x+3)-6x^{1/2}ln(x^2-x)$ and then used the log rules for derivatives I know and the product rule on the right side and wound up with $$y'=y[\frac{18}{2x+3}-\frac{3ln(x^2-x)}{x^{1/2}}+\frac{6x^{1/2}(2x-1)}{x^2-x}]$$ The 18 over 2x+...

0
Q: convergence rate of forward backward operator splitting algorithms

MridulaI am looking for some latest material on convergence rate of the basic forward backward operator splitting algorithm. After googling, I found the following: http://epubs.siam.org/doi/abs/10.1137/S1052623495290179 http://link.springer.com/chapter/10.1007/978-1-4419-9569-8_10#page-1 I am newbie...

Welcome to Math.SE, Mridula. Consider replacing (analysis) with a more specific tag for the relevant branch of analysis. (autocomment)Normal Human 20 secs ago
0
Q: How to get limit on integration for a convolution of two density functions

LindaKFor two density functions: Suppose again that Z = X + Y . Find fZ(z) if fX(x) = fY(x) = { x/2, if 0 < x < 2, { 0, otherwise I understand this is a convolution fX *fY but I don't understand how to obtain the limits of integration since they are not -inf to +inf. X = x Y = Z - x fZ(x) = i...

This site uses MathJax formatting of formulas. More tips here. (from a bot)Normal Human 20 secs ago
0
Q: $x\in \mathbb{R}>0 \rightarrow \exists y\in \mathbb{R}>1 s.t. xy>1$

JabernetI am going through my practice sheet before finals in my proofs class, and I would like critique, please. My understanding of the If-Then and the "hidden implications" is that each real number $x>0$ has at least one $y>1$ such that $xy>1$. If this is the case, then it is true, and here is my fo...

A title should not be all-MathJax; having some plain text helps with search and navigation. Tag (proof-verification) should not be the only tag a question has. Please add a tag for a subject area to which the question belongs. (from a bot)Normal Human 20 secs ago
0
Q: Concave functions

user101388Given $(X,\mathcal{F},m)$ a measure space with $m(X)=1$ and $||f||_p<\infty$ for some $p>0$. Need to show that $\forall q \in (0,p)$ $$\int \log |f| dm\leq \log(||f||_q),$$ $$ \log ||f||_q \leq \frac{\int |f|^qdm -1}{q},$$ $$\lim_{q\to 0} \frac{\int |f|^q dm -1}{q}=\int \log |f| dm$$ and finall...

Short title. Concave functions
0
Q: prove that this function is not increasing

Maths beginnerenter image description here This is from Abbott's understanding analysis, but I can't really figure it out

 
4:23 AM
2
Q: Is [visual] tag too broad?

RaptorThe visual could mean a lot of things. Is it too broad to use?

 
0
Q: Biomath question

IDidntDoThatMicrotubules play a role in the migration of chromosomes to opposite ends of a mitosing cell during anaphase. Microtubules are hollow tubes 24 to 25 nm in diameter composed of 13 parallel rows. The parallel rows are called protofilaments and are made of heterodimers called tubulin subunits. A tub...

Words such as question are uninformative in titles. Please edit the title so that it better describes the specifics of your question. Do not hesitate to make it longer or include a formula if needed. More tips here. (autocomment)Normal Human 20 secs ago
0
Q: i need help, this makes no sense

johnSo when I graph 2x - y = 6 and x + 2y = -2, I see them intersecting at points (2,-2). HOWEVER, when I set them equal to each other (2x - 6 = -1/2x -1) I don't get 2 for x. Can someone please clarify how to do this?

Welcome to Math.SE, john. Words such as help do not add information to titles. Please edit the title so that it better describes the specifics of your question. Do not hesitate to make it longer or include a formula if needed. More tips here. (autocomment)Normal Human 21 secs ago
 
4:44 AM
0
Q: $\displaystyle \lim x^2 = a^2$ as $x$ goes to $a$

user276387 Prove that $\displaystyle \lim_{x \to a} x^2 = a^2$ Let $\epsilon > 0$, and let $\delta = \min(\frac{\epsilon}{2|a|+1}, 1)$. Suppose $x \in\mathbb{R} - \left\{a\right\} $ and $|x-a| < \delta.$ Then $|x-a| < 1$ which implies $ -1 < x-a <1 $ which implies $ a-1 < x < a+1$ which implies $2a-1< ...

Tall formulas in titles break the layout of question lists. Please remove \displaystyle in the title. (from a bot)Normal Human 21 secs ago
0
Q: How to show that a continuous function has a fixed point

Maths beginnerSuppose that $a < b$ and $f: [a,b] \to\mathbb{R}$ is a continuous function such that the range of $f$ contains $[a,b]$. Prove that $f$ has a fixed point.

0
Q: Levenshtein distance calculations

Andrew HedgesWhat is the correct Levenshtein distance between the following strings? hahaha ahahah These sites report different values: http://andrew.hedges.name/experiments/levenshtein/ (6) http://planetcalc.com/1721/ (2) The first site seems to be saying that you must compare each letter in place and ...

0
Q: Increasing/Decreasing intervals

Richard ChangWhy do some people used closed intervals when referring to those intervals? Concavity uses open intervals, so why does increase/decrease use closed intervals?

Short title. Short question. Increasing/Decreasing intervals
0
Q: The name $\mathcal{C}^\omega$

user173897Let A be a set (of real numbers); define $\mathcal{C}^\omega (A)$ as the set of all real-valued functions that are defined, bounded, and analytic on A. My question is simply this: how did $\mathcal{C}^\omega (A)$ get its name? Who named it? What does $\omega$ mean in this context? How does it...

 
5:04 AM
0
Q: Can a modrator of any stack exchange sites close my question as Offtopic on stackoverflow?

StarkeenCan a modrator of any stack exchange sites close my question as Offtopic on stackoverflow? forex.. Modrater of http://spanish.stackexchange.com or other stack sites has 20k or more reputations on stackoverflow but he is not a modrator.

 
0
Q: Why should I learn mathematics furthermore?

Angelo MarkHow can it be useful if I have a job which involve no mathematics. Some times it is the ugly truth we face. We learn mathematics , and we get a job which does not require anything what we have learnt. Any ideas ?

 
0
Q: Can a modrator with less then 20k reputation close my question as duplicate?

StarkeenCan a modrator with less then 20k reputation close my question as duplicate or offtopic? for example a modrator with 1k or more reps.

 
5:25 AM
0
Q: Find the limit of e^(x^2) / xe^x

Steve PPlease help me find this limit. I have already tried to use L'Hopitals rule and logarithmic differentiation but it turns into a quadratic.

Short question. Question contains please. Find the limit of e^(x^2) / xe^x
 
-1
Q: Should Tau be an alias or a separate tag?

LamonteCristoI'm asking for my own education, since I don't understand the inherent benefits of Tau. Is there a legitimate need to tag questions "tau" or is "pi" sufficient? Should TAU be an alias to Pi? ... or a separate Tag?

 
0
Q: Unsure how to treat y in this derivative/log problem

windy401$h(y)= \ln(y^2 \cos y)$ Treating it like a normal variable like an x isn't working for me, the way we used y's in earlier problems where you get a y' in there doesn't seem right, so I'm not quite sure here.

0
Q: Could someone check my solution for finding constant of a difference quotient?

RooneySo the question was, Let $f: \mathbb{R} \rightarrow \mathbb{R}$ be three times differentiable and $f'''$ is bounded, find constants $a,b,c$ such that $$f''(x) = \lim_{h\rightarrow 0} \frac{af(x-h)+bf(x)+cf(x+2h)}{h^2}$$ My Solution: Expand $f (x-h)$ and $f(x+2h)$ by Taylors theorem and solve...

0
Q: complex integration 5

Olya12$\int\limits_{0}^{2\pi}e^{cos\varphi}(cos\varphi-sin\varphi)d\varphi$ I think $e^{i\varphi}=z$ $\to d\varphi=\frac{dz}{iz}$ $cos\varphi=\frac{z^2+1}{2z}$ $sin\varphi=\frac{z^2-1}{2iz}$ $\oint\limits_{|z|=1}^{}e^{\frac{z^2+1}{2z}}(\frac{z^2+1}{2z}-\frac{z^2-1}{2iz})dz$ $z=0$ - essential ...

Short title. Title ends with a digit. complex integration 5
 
0
Q: Blacklist "Muscle" on Super User

SathyaThe body building spam seems to be relentless and while lot more is blocked, we still see close to 4-5 posts every day about this. A search for "muscle" indicates that so far there have been 445+ such posts. Can we add muscle to the blacklist?

 
5:50 AM
0
Q: Basis of Kernel of Linear Transformation

Guy1234567T: M33 -> M33 defined by T(A) = 1/2(A+A^T) What is the basis for the kernel of this linear transformation? So how do I solve this?

0
Q: Combinatorics: Is this utilizing the pidgeon-hole principle?

Onikouzou How many ways can we assign four different jobs to five different employees, assuming it is possible to assign more than one job to any employee?

0
Q: Non Linear Initial Boundary value problem solution. Image attached

GabrielCan some one please help me on the attached image. Is it possible to user Fourier series to find the solution or is there any other way? Assuming g(x,t) = Summation n = 1 to infinity Sin(nx/L) , what could be the possible solution.

Consider adding a tag for a broader subject area to which the question belongs. Some of these tags might fit. (autocomment)Normal Human 21 secs ago
0
Q: On proving $\lim \frac{n^2}{n^2+n+1} = 1$

user276387 Prove that $\lim \frac{n^2}{n^2+n+1} = 1$ Let $\varepsilon > 0$ and let $N = \frac{1}{\varepsilon}.$ Then $n > N$ implies $n > \frac{1}{\varepsilon} \implies \frac{1}{n} < \varepsilon.$ But $\displaystyle \frac{1}{n} = \frac{n+1}{n(n+1)} = \frac{n+1}{n^2+n} > \frac{n+1}{n^2+n+1} = \bigg|\fr...

Tagged proof-verification. On proving $\lim \frac{n^2}{n^2+n+1} = 1$
0
Q: Solving For X In A 4th Degree Polynomial

Imagine DragonsThe examples they use on my book are basically from basic arithmetic like 1, 2 and 3 to calculus and calculating derivatives, which is really annoying because I can't build the fundamental skill required to do harder questions. Anyhow, I have to solve an non-factorable inequality. Normally, I wou...

This site uses MathJax formatting of formulas. More tips here. (autocomment)Normal Human 21 secs ago
0
Q: How to find the domain to each petal (Polar Graphing)

Kei HarukiGiven the equation: r=4cos3(theta)...how can I find the domain of each petal? Help!

 
6:22 AM
2
Q: Would this type of question be on topic?

AlwaysLearningNewStuffINTRODUCTION AND RELEVANT INFORMATION: I have a desktop application with a window that needs to display large amount of entry fields. Please take a look at below image: As indicated on the image, controls in the red rectangle should be in main window, other ones can be shown when needed, usin...

 
6:47 AM
0
Q: Beginner Troubleshooting Questions

IkeRelated topics: How to answer extreme beginner questions Are beginners' questions welcome? How do people feel about beginner troubleshooting questions? Note that I'm not asking about beginner Qs, I'm not asking about troubleshooting Qs. I'm asking about the precise combo of these two things. ...

 
0
Q: Solve the definite integral given the identities?

Steve PHere is a picture of the textbook question cause I'm not completely sure how to format it. I don't know how the answer for d) is 10. https://gyazo.com/1fff6acd3b0076a313aacfd119743d21

0
Q: Modulo problem : the problem of the uniqueness of the result.

wawarjiHow to prove in the set of $A$ = {1, 2, 3, ..., 2n-1}, when starting with any $i$ element $A$, if $i + n$ mod $(2n-1)$ as many as $2n-1$ times where the new result is added also by $n$ then all of the elements of A become the results of the operation or in other words all of the results are diffe...

0
Q: Application of Chern class

VincentI am taking a course about differentiable manifold and I need to prepare a representation about Chern class. Although now I am familiar with the properties of Chern class, but I can not find good examples of the applications of Chern class. As a topological invariant of vector bundle, I really w...

0
Q: the residue at the singular point

Olya12We need to find residue $\frac{1}{cos^2z}$ $cosz=0$ $z=\frac{\pi}{2}+\pi k$ - order 2 poles as the next?

 
7:23 AM
0
Q: Yoneda's Lemma in Vakil's notes

Jesse Solomon Scottenter image description here in the exercise 1.3Y, what does 1.3.10.2 commutes with the maps 1.3.10.1 mean? I can't see any relation between 1.3.10.1 and 1.3.10.2.

0
Q: What's the set $\mathbb{N}_0$?

mavaviljIn some papers I've seen the following set: $$\mathbb{N}_0$$ What's the set $\mathbb{N}_0$?

0
Q: Supremum over ellipsoid set

aberdyshIn Boyd's Convex Optimization Textbook, page 157, it is stated: $ \mathrm{sup}\{a_i^\top x\; |\; a_i\in\mathcal{E}_i \} = \bar a^\top x + \mathrm{sup}\{u^\top P_i^\top x\; |\; \lVert u \rVert_2 \leq 1 \} = \bar a^\top x + \lVert P_i^\top x \rVert_2 $ But I do not see how $\mathrm{sup}\{u^\top...

0
Q: Let $f:[-1,\infty]\to R$,$f(x)=(x+1)^2-1$.Then the solution set of the equation $f^{-1}(x)=x$ is $\left\{-1,0\right\}$

diyaLet $f:[-1,\infty]\to R$,$f(x)=(x+1)^2-1$.Then the solution set of the equation $f^{-1}(x)=x$ is $\left\{-1,0\right\}$.Is this statement true of false? $f(x)=(x+1)^2-1\Rightarrow f^{-1}(x)=\sqrt{x+1}-1$ Now solving $f^{-1}(x)=x$,we get $\sqrt{x+1}-1=x$ $\sqrt{x+1}=x+1$ $x=0,-1$ It appears that...

Question contains please. [Let $f:[-1,\infty]\to R$,$f(x)=(x+1)^2-1$.Then the solution set of the equation $f^{-1}(x)=x$ is $\left\{-1,0\right\}$](math.stackexchange.com/q/1568944)
0
Q: Successful studying for a proof-based course final exam?

playitrightI'm currently taking a Transition to Advanced Mathematics course, which is entirely proof-based, so it's pretty new territory. Up until now, all the classes I've taken were fairly computational, so studying was just doing practice problems. I've already made flash cards of all the definitions (wh...

0
Q: Probability in combinatorics

DibsHere is the problem. I have 3 marbles in a bag, a red one, a blue one and a green one. Let's say you win a prize if you pick the red marble and let's say that you can pick two times in the bag and each time you have to put the marble back in the bag. Here are then the possible results if you say ...

 
7:59 AM
0
Q: Smaller radius of convergence

EliminationSuppose we have two power series, with coefficients $a_n, b_n$, both with radius of convergence, $R$. Let the power series with the coefficients $a_n+b_n$, with radius $R'$ I need an example for scenario where $R'<R$. The other cases are easy NY the way. Written from mobile. Thanks in advanc...

0
Q: pdf of S1 given X(t)=n where X(t) is a poisson process and S1 the time of first event

Jake LeeLet {X(t)~Pois(λt)} be a Poisson process with parameter λ, and 0 How do can find the pdf of S1|X(t)=n and use it to find E[S1|X(t)=n]. Also, what is the pdf of Si|X(t)=n for i=1....n? what would be E[S1+S2+...+Sn|X(t)=n]

Questions tend to get more attention when they have a tag for a broad area of mathematics relevant to the question. Some of these tags might fit. (from a bot)Normal Human 21 secs ago
 
0
Q: Split off the [string-split] tag

Maroun MarounSomeone can do many operations on a string, one of them is split. I don't understand why we should have the string-split tag. Having it would legitimate creating "count-string", "replace-string" and many other "operation-string" tags. I think that string and split are enough to indicate that th...

 
0
Q: Question on geometry triangle and incenter

AyushakjLet ABC be a triangle. Let B' and C' denotes respectively the reflection of B and C in the internal angle bisectors of angle A. How do I prove that the triangles ABC and AB'C' have same incenter.

0
Q: Orthonormalize sequence

MarussTI am having difficulty -normalizing sequence $1,x,x^3...$ in $L^2([-1,0]\cup[1,2],dx)$ So I begin by choosing basis $1,x,x^2...$ and i need to use Gram-Schmidt? or how should i approach this? I have attempted to integrate x from -1 to 0 and add integral from 1 to 2, but I don't think I understand...

Words such as question do not add information to titles. Please edit the title so that it better describes the specifics of your question. Do not hesitate to make it longer or include a formula if needed. More tips here. (from a bot)Normal Human 21 secs ago
0
Q: Should the performance of a PID controller be independent of the input?

MWijnandAssume you designed a PID controller to let a given system track a unit step. Will the controlled system exhibit the same behaviour with regard to step inputs with different amplitudes?

 
8:26 AM
0
Q: are these Graphic Sequence?

Shinning EyesHello I am trying to solve this but i am confuse how can do that. Are these sequences are graphic? Explain your answer. a. 4, 3, 3, 2, 1. b. 6, 3, 2, 2, 1, 0.

Short title. Short question. are these Graphic Sequence?
0
Q: $E \subset [0,2\pi]$ has positive measure, for any sequence $t_n$ of real numbers, $\lim_{n \to \infty} \int_E \cos(n x + t_n)\,dx = 0?$

user2503162Assume that $E \subset [0, 2\pi]$ has positive measure. For any sequence $t_n$ of real numbers, do we have$$\lim_{n \to \infty} \int_E \cos(n x + t_n)\,dx = 0?$$

Short question. [$E \subset [0,2\pi]$ has positive measure, for any sequence $t_n$ of real numbers, $\lim_{n \to \infty} \int_E \cos(n x + t_n)\,dx = 0?$](math.stackexchange.com/q/1568987)
0
Q: Find the variance of Y .

TheValarsI have conflicting answers with different methods. Here is the question: The number of defects per yard in a certain fabric, Y , was known to have a Poisson distribution with parameter $\lambda$. The parameter $\lambda$ was assumed to be a random variable with a density function given by: $f (...

0
Q: General term of the recurrence

user249117The recurrence is given as follows :- f(i) = f(i-4) + floor (((i + 2)(i + 6)/12)^2) ( ^ symbol stands for raised to power ) f(0)=1 , f(1)=3, f(2)=7 , f(3)=14 I tried finding its general term but failed miserably . Can someone help in finding its general term ?

This site uses MathJax formatting of formulas. More tips here. (autocomment)Normal Human 21 secs ago
0
Q: Is there a better way of solving this problem other than guessing what the 2x2 matrices are?

User001This is an old exam question from linear algebra that I am working on, The problem statement is: Part 1 Can you find $2×2$ matrices A,B, and C, so that $A^2≠0$, $B^2≠0$ and $C^2≠0$ but AB=0,BC=0 and CA=0? Part 2 Can you find $2×2$ matrices A,B, and C, so that $A^2≠0$, $B^2≠0$ and $C^2≠0$ ...

0
Q: Basics of bipartite graph

Shinning EyesAnyone help me to solve this: $K_{i,j}$ denotes a complete bipartite graph of (i, j) vertices. A Ki is a complete undirected graph of n vertices. a. How many vertices are there in a $K_{i,j}$? b. How many edges are there in the graph ? i*j c. What is the total degree of the graph...

Short title. Title contains basic. Basics of bipartite graph
0
Q: Find value of a in following number theory question

AyushakjHow do I find all real numbers a such that 4 < a < 5 and a(a-3{a}) is an integer. here {a} is an fractional part of a.

Words such as question do not add information to titles. Please edit the title so that it better describes the specifics of your question. Do not hesitate to make it longer or include a formula if needed. More tips here. (autocomment)Normal Human 21 secs ago
1
Q: Subgroups and Identity

Jane CraftsmanLet H and K be a finite subgroups of a group G whose orders are relatively prime, that is, gcd(|H|,|K|) = 1. Show that H ∩K={e}, where e is the identity in G. I really have a very small knowledge regarding abstract algebra, since I've only explored this topic today, and I have to answer this bef...

0
Q: Correlation between variables

user297544If corr(X,Y)=0.9, will there exist a third variable Z such that Corr(X,Z)=corr(Y,Z)=-0.9? I tried reasoning out there is one such variable using the regression equation Y=a+bX but reached a dead end.

Welcome to Math.SE, user297544. Consider adding a tag for a broader subject area to which the question belongs. Some of these tags might fit. (autocomment)Normal Human 21 secs ago
 
9:07 AM
0
Q: Acts of sabotage on MO?

TatendaI have noted with great concern from my posts (and those of other newer users) that, certain people seem to be downvoting other people's questions for no apparent reason at all. I highlighted this worrying fact in one of my comments and fortunately the platform moderators assisted. But in my rece...

0
Q: What happens to Bounty when the bounty period expires (including the 24 hour grace period)

Program-Me-RevWhat happens to Bounty issued on a Question on StackOverflow when the bounty period expires (including the 24 hour grace period), but the question has answers, but non of them satisfactory, and non has a score of more than 2, and the person who issued the bounty has not accepted any answer?

0
Q: What's wrong with the code formatter

Charlie HI have been trying to format my code with the code formatter in the following answer: Handle Page load time angularjs The first code snippet doesn't get formatted at all. I needed to include some html tags in the second snippet. But they won't show. Trying to format seem to take more time th...

 
0
Q: PID tuning for system with parameter

MWijnandHow can I tune a PID to control a system with a parameter? Can I know beforehand how to change the PID parameters in function of the value of the system parameter?

0
Q: Clifford semigroups!

LolaIs a subsemigroup of a Clifford semigroup also a Clifford semigroup? I can prove that the maximal group image of a Clifford semigroup is a Clifford semigroup, but I am not sure whether any subsemigroup is also a Clifford semigroup!!

Short title. Clifford semigroups!
 
9:35 AM
0
Q: is this graph follows Hamiltonian path and Eulerian path?

Aadnan Farooq AFor this following graph Hamiltonian path and Eulerian path exist or not. Basic definition A cycle that uses every vertex in a graph exactly once is called a Hamilton cycle, and a path that uses every vertex in a graph exactly once is called a Hamilton path. So I think it follows Hamilton path,...

Questions tend to get more attention when they have a tag for a broad area of mathematics relevant to the question. Some of these tags might fit. (autocomment)Normal Human 21 secs ago
0
Q: Evaluating contour integral of complex conjugate

J SimmonsThis is part of a homework assignment. Any hints will be useful, I haven't made any progress. I need to evaluate: $\int_{|z-1|=1} \bar{z}^n dz, n \in \mathbb{Z}$

0
Q: Optimal strategy for unlocking Cho'gall (probability intuition question)

user1178598Right now there is an event occurring in Heroes of the Storm where a special hero (Cho'gall) is unlocked if you play with another player currently playing that hero. I ran into a bit of an intuition problem when I started thinking about the optimal strategy for attempting to unlock this hero. ...

0
Q: If $f$ is a one-one mapping from set $A$ to set $A$,then $f$ is onto.

Vinod Kumar PuniaState true or false $(1)$If $f$ is a one-one mapping from set $A$ to set $A$,then $f$ is onto. $(2)$If $f$ is an onto mapping from set $A$ to set $A$,then $f$ is one-one. I do not understand whether it is true or false.If true,why true.If false,why false.No concrete reasoning,example and count...

 
-2
Q: Change the topic about SuperUser on off-topic close reason

Pandya At the Off-Topic Close reason, we use "those about Windows on Super User". So, there is full chance of misunderstanding that "SuperUser is about Windows power user"! But actually SuperUser welcomes the questions about computer software and hardware (in general). So, I request to change...

0
Q: Is there any way to get more top users for a tag?

Salvador DaliFor each tag one can see a list of top users. For example here is a list for javascript. Is there any way to get a bigger list may (be with an information about the number of positive answers each of them wrote in last X days)?

 
10:14 AM
0
Q: Vector perpendicular to a quaternion

Dénes Ákos NagySorry if my question is really basic, I'm quite new to the field of quaternions. I face a problem where I would like to find a vector perpendicular to a quaternion. I know how to find an arbitrary vector perpendicular to a vector, but I would like to select the one pointing in the direction of th...

Welcome to Math.SE, Dénes Ákos Nagy. Questions tend to get more attention when they have a tag for a broad area of mathematics relevant to the question. Some of these tags might fit. (autocomment)Normal Human 33 secs ago
 
10:27 AM
0
Q: how can i Prove that by adding one edge to G you create a cycle in G?

Aadnan Farooq AAny one help me to show the prove for this? Let the undirected graph G = (V, E) be a tree. Prove that by adding one edge to G you create a cycle in G.

0
Q: Symmetric power of tautological representation of $U(n)$

user2715119Let $S^kV$ be the $k$-th symmetric power of tautological representation of $U(n)$ how to see that it's irreducible? I'm trying to do it using weight, but with no benefits..

0
Q: Regularity of a language

lplouis$$L = \{w|w \space does \space not \space contain \space 000\}$$ $$L2 = \{w | xwy \in L \space for \space some \space x,y \in \space (0+1)^*\}$$ Is L2 regular? I am thinking regular language is closed under concatenation, but it seems that this language should be irregular. And I have no idea how...

0
Q: Level sets and boundary

SashaI consider level sets $E_t = \{u <t\}$ and $N_t = \partial E_t$ in a manifold $M$ on dimension 3. Is this equality true : $\int_{E_t}|\nabla u| = |\partial E_t|$ ? Thanks

Short title. Short question. Level sets and boundary
0
Q: how to slove these with gamma and beta function thankss

user297497I can't solve this, no matter how I try

0
Q: Proving identity of Markov chain.

T-bjornI am in a need of proving the following identity. Hope someone can help me. Let {x_n, n=>0} be a homogenous Markov Chain. Show that P(X_{n+1}=k_1,...,X_{n+m}=k_m (conditiong bar) X_0=i_0,...,X_n=i)=P(X_1=k_1,...,X_m=k_m (conditiong bar) X_0=i) I know I should use multiple conditiong and the Marko...

Welcome to Math.SE, T-bjorn. This site uses MathJax formatting of formulas. More tips here. (from a bot)Normal Human 29 secs ago
0
Q: Help me with the result of this determinant..

A6ETF$$ D = \begin{vmatrix} 1 & 1 & 1 & \dots & 1 & 1 \\ 2 & 1 & 1 & \dots & 1 & 0 \\ 3 & 1 & 1 & \dots & 0 & 0 \\ \vdots & \vdots & \vdots &\ddots & \vdots & \vdots \\ n-1 & 1 & 0 & \dots & 0 & 0 \\ n & 0 & 0 & \dots & 0 & 0 \\ \end{vmatrix} =n*1*(-1)^\frac{n(n-1)}{2} $$ I don't quite understand th...

Words such as help are uninformative in titles. Please edit the title so that it better describes the specifics of your question. Do not hesitate to make it longer or include a formula if needed. More tips here. (autocomment)Normal Human 23 secs ago
0
Q: group actions on spheres

R. S.Let $\mathbb{Z}/2$ act on the $m$-sphere $S^m$. If the action is not trivial, can we conclude that the action is homotopy equivalent to the antipodal action? That is, the following diagram commutes up to homotopy? Can we generalize this to $S^1$-actions on $S^{2m+1}$ and $S^3$-actions on $S^{...

 
11:14 AM
-1
Q: Illuminator vs Copy Editor

Tim BI was browsing the badges earlier and I noticed something a little...strange. Copy Editor Requires you to edit 500 posts. Illuminator Requires you to edit and answer 500 posts. Unless I've missed anything it's completely impossible to get Illuminator without also getting Copy Editor. This is d...

 
0
Q: Solve matrix vector equation

guest4890132Let $A$ be a real $n\times n$ matrix and $w,x$ real $n\times 1$ vectors. For fixed $A$ and $w$ solve the following for $x$: $(x^\top A x)w - (x^\top w) (A+A^\top) x = 0$ Any hints? I do not really feel confident on what kind of rearrangements are legit since multiplication is generally not com...

0
Q: If $X\sim N(\mu_x,\sigma_x^2)$ and $Y\sim N(\mu_y,\sigma_y^2)$ what is $\text{Var}{(X/Y)}$

FreemanIf $X\sim N(\mu_x,\sigma_x^2)$ and $Y\sim N(\mu_y,\sigma_y^2)$ what is $$\text{Var}{(X/Y)}$$

0
Q: What is wrong with this proof of chain rule?

luka5z Let $U$ and $V$ be open subsets of $\mathbb{R}$. Let $f:U\rightarrow V$ be differentiable at $x_0$ and $g:V\rightarrow\mathbb{R}$ be differentiable at $f(x_0)$. Then $g\circ f$ is differentiable at $x_0$. What is wrong with the following "proof" of the above fact: $$(g\circ f)'(x)=\lim_...

Title contains wrong. Tagged proof-verification. What is wrong with this proof of chain rule?
 
0
Q: How to remove redirect from old username to new username

daredevilRecently I have changed my username on all stackexchange sites but it still shows up on search engines. Upon clicking, stackoverflow/exchange redirects me to the new username. Is it possible to remove the link between the old username and the new one and keep my points ? Reason : privacy. Re...

 
11:33 AM
0
Q: Minimum Value Of a Complex quantity

botLet $Z_1,Z_2,Z_3,Z_4 \in\mathbb{C}$ such that $Z_1+Z_2+Z_3+Z_4=0$;$|Z_1|^2+|Z_2|^2+|Z_3|^2+|Z_4|^2=1$ then the minimum value of $|Z_1-Z_2|^2+|Z_2-Z_3|^2+|Z_3-Z_4|^2+|Z_4-Z_1|^2$ is ?

 
0
Q: Is "look into x" an acceptable answer?

T JRecently I flagged this answer as Not an Answer, but it was declined mentioning that flags should not be used to indicate technical inaccuracies, or an altogether wrong answer. The answer contains the following: Look into min-height I didn't flag it based on any technical inaccuracy, ...

0
Q: Why was this question marked as duplicate?

Anchit VirmaniI asked the following question : Modified version : How many subsets XOR to a given value? but it was marked duplicate. The original question was to find how many subsets of a given set XOR to a particular value. But my question was : To find how many subsets of a given mul...

 
0
Q: Binomial argument in Euler's Transformation

Ole PetersenPage 20 http://www.cs.umb.edu/~offner/files/pi.pdf he says that "We see immediately that we have $\Delta^{k}a_{m} = \sum (-1)^{j} (\frac{k}{j})x_{j}$." Why exactly is that? There must be a more formal way to prove this in mathematical terms.

0
Q: Family of circles, maybe?

Omkar Kulkarni Circles are drawn passing through the origin O to intersect the coordinate axes at points P and Q such that $(m)(OP)+(n)(OQ)=k$, then the fixed point satisfying all such circles is? (A) $\left(m,n\right)$ (B) $\left(\dfrac{m^2}{k},~\dfrac{n^2}{k}\right)$ (C) $\left(\dfrac{mk}{m^2+n^2...

0
Q: Path Independence Vectors

the manIf I wanted to know whether a line integral $\int_C f \cdot d\textbf{r}$, where $d\textbf{r}$ is $(dx,dy),$ is independent of path, what is the best way of doing this? I know it is path independent $\iff$ it's conservative. So I could show the $\nabla \times f=0$, I could find two different paths...

0
Q: every topological space can be realized as the quotient of some Hausdorff space...

user297564How can i prove that every topological space can be realized az the quotient of some hausdorff space?! I tried to show this by using the intersection of two open sets in x(for f:z-->x)

0
Q: How to get $||u_m(0)||_{H_0^1(U)}\le||g||_{H_0^1(U)}$?

lanse7ptyWhen I read Evans' PDE, I want to use the Theorem 2 in picture below to get $||u_m(0)||_{H_0^1(U)}\le||g||_{H_0^1(U)}$. So ,I let the $T=0$, then I get $$ ||u_m(0)||_{L^2(U)}\le C||g||_{L^2(U)} $$ There is a little difference , I don't how to deal it .The question is from the 362 of Evans' PDE, ...

Tagged pde, differential-equations. How to get $||u_m(0)||_{H_0^1(U)}\le||g||_{H_0^1(U)}$?
0
Q: how to implement a user-defined criteria in matlab ode solvers?

melodyis there a way to add a user defined convergence criteria to an ode solver so that the solution is stopped? I am using the solvers in to solve fluid dynamics problems and I am not sure whether the already used tolerances are sufficient. I wold like to have some control over the convergence crite...

0
Q: Convolution in $L^p$

MatrizI'm trying to solve the following task: Let $f:\mathbb{R}\rightarrow \mathbb{R}$ is measurable and $h\in\mathbb{R}$, we define the measurable function $\tau_hf:x\rightarrow f(x+h)$. Let $1\leq p<q\leq \infty$ such that $\frac{1}{p}+\frac{1}{q}=1,f\in L^P(\mathbb{R}),\ g\in L^q(\mathbb{R})$. Reca...

Short title. Convolution in L^p
0
Q: Question about sum of residue classes

Khadija MbarkiIs there a sharper bound for the following sum $$S:=\sum_{d \in (Z/qZ)^{*}} \overline{d},$$ where $a$ is any integer and $\overline{d}$ is the inverse of $d$ modulo $q>0$? Thanks in advance.

Words such as question do not add information to titles. Please edit the title so that it better describes the specifics of your question. Do not hesitate to make it longer or include a formula if needed. More tips here. (autocomment)Normal Human 59 secs ago
0
Q: A question about conditional probability on drawing cards

JackyIn a game of drawing cards, $4$ cards will be drawn from the deck randomly. In the deck of cards, $0.4$ of them are black, $0.6$ of them are white. Among the black cards, $20\%$ of them are worth $1$ point, $50\%$ of them are worth $0$, and $30\%$ of them are worth $-1$ point. Among the white car...

Words such as question are uninformative in titles. Please edit the title so that it better describes the specifics of your question. Do not hesitate to make it longer or include a formula if needed. More tips here. (autocomment)Normal Human 25 secs ago
0
Q: Dense set in the unit ball- reference needed

martinFor $x \notin \pi\mathbb Q$, that is, a real $x$ that is not a rational multiple of $\pi$, consider the set $$\{(\cos nx,\sin nx):n = 0,1,2,...\}.$$ It is known that this set is dense in the unit ball $B(0,1)$ of $\mathbb R^2$. Could someone please give me a proof or reference for a proof?

0
Q: Graph Picture for specified vertices and edges

Aadnan Farooq APlease, anyone, help me to tell is it correct? Draw the picture of the specified graph (including any isolated vertices): Graph H has vertex set {v1,v2,v3,v4,v5} and edge set {e1,e2,e3,e4} with edge-endpoint function as follows: Edge Endpoints e1 {v1, v4} e2 {v1, v2} e3 {v1, v...

 
12:22 PM
0
Q: limit question realted to exp functions

NimaWhat is lim(1/x)^sinx when x goes to zero?

Welcome to Math.SE, Nima. Words such as question are uninformative in titles. Please edit the title so that it better describes the specifics of your question. Do not hesitate to make it longer or include a formula if needed. More tips here. (from a bot)Normal Human 21 secs ago
0
Q: Please help me with this off topic question

AyushakjHow do I write perfect mathematical symbols on this site. I have problem to write perfectly with all symbols placed correctly.

Words such as please, help, question are uninformative in titles. Please edit the title so that it better describes the specifics of your question. Do not hesitate to make it longer or include a formula if needed. More tips here. (autocomment)Normal Human 21 secs ago
0
Q: I can't write $\frac {1}{1+z^2}$ as a power series centered at $z_0=1$ [Help!]

SymphonizedI'm trying to solve a question where I need to write $\frac {1}{1+z^2} $ as a power series centered at $z_0=1$ I can't use taylor expansion. So my first thought was to rewrite the function in a form where I can apply the basic identity: $$ \frac{1}{1-x} = \sum_{n=0}^\infty x^n $$ So let's rewr...

Title contains help. [I can't write $\frac {1}{1+z^2}$ as a power series centered at $z_0=1$ [Help!]](math.stackexchange.com/q/1569168)
0
Q: $L^2$ mapping is necessarily onto or not?

Albert GeeFor $f \in L^2(\mathbb{R})$, let $$Tf(x) := \int_0^1 f(x+y)\,dy.$$Do we necessarily have that$$S: L^2(\mathbb{R}) \to L^2(\mathbb{R}),\text{ }Sf = f - Tf$$is onto?

0
Q: Sylvester's criterion for negative semidefinite matrices

KalIs there a Sylvester's criterion for negative semidefinite matrices? I suspect such a criterion to be: All principle minors with odd dimension are non-positive. All principle minors with even dimension are non-negative. Please include a reference if it exists.

 
12:38 PM
0
Q: Stackoverflow "contact us" option "view ticket"

ferrybigI just submitted yet another website that is cloning StackOverflow. After I got a replay in my mailbox that stackoverflow was working on it, I discovered that gmail sees that the mail comes from zendesk and has an associated ticket id. Button redirects to: https://stackexchange.zendesk.com/re...

 
0
Q: Is there a shorter proof to show that this complex intergral is constant?

SevenstarsI have the integral, $$I(R) = \int_{C_R}\frac{1}{z(z-1)^2} dz$$ with the property that $$\left|\frac{1}{z(z-1)^2}\right| \leq \frac{1}{R(R-1)^2} \quad |z|=R>1$$ Where $C_r$ is the contour defined by the circle of radius $R>1$ so $C_r $ can be parametrized as $z(\theta)=Re^{i\theta}$ The goal...

0
Q: Finitely-generated $ \mathbb R[x]$-module

NatalieShow that ${\mathbb R[x]}/(x^4−1)$ is a finitely generated $\mathbb R[x]$-module. I know that $ \mathbb R[x]$ is a PID, but I'm not sure where to go from there.

Short question. [Finitely-generated $ \mathbb R[x]$-module](math.stackexchange.com/q/1569179)
0
Q: P(A|C)=P(A|B)*P(B|C)?

whoisitIs this formula P(A|C)=P(A|B)*P(B|C) correct according to Bayes' theorem?

Consider adding a tag for a broader subject area to which the question belongs. Some of these tags might fit. (autocomment)Normal Human 21 secs ago
0
Q: Product of tensor with vector

user143763$$ \begin{align} \left(v_1^{\mathrm T}\cdot A\cdot v_2\right)\cdot\left(B\cdot v_3\right)\stackrel{?}{=}&v_3\cdot\left(\begin{pmatrix}v_1^{\mathrm T} & v_1^{\mathrm T} & v_1^{\mathrm T}\\v_1^{\mathrm T} & v_1^{\mathrm T} & v_1^{\mathrm T}\\v_1^{\mathrm T} & v_1^{\mathrm T} & v_1^{\mathrm T}\end{p...

0
Q: Optimization problem with sum over integrals

FooBarI'm looking at the following optimization problem: $$ max_{t\in(0, T)} Y(t) = \sum_{s=1}^\infty P(s, a t) d(T-t)\int g(x) h(x; s) dx$$ where $h()$ is given by $$H(x; s) = Prob(\min \leq x) = 1 - \prod_{i=1}^s Prob(x_i \geq x) = 1 - \prod_{i=1}^s(1-F(x))\\ h(x; s) \equiv \frac{\partial H(x; s)}...

 
12:54 PM
0
Q: MathJax rendering \bk differently in question display and edit modes

Emilio PisantyI'm completely stumped by this. The question Enegy Expectation Value uses the LaTeX command \bk, which is definitely not a standard LaTeX command. Quite fittingly, it fails to display correctly on the question page. Seeing this, I clicked on the edit button to fix that LaTeX, but to my great s...

 
0
Q: Question on quadratic polynomials with real cofficients

AyushakjLet P(x) = x^{2} ax + b be a quadratic polynomial with real coefficients. Suppose there are real number s ≠ t such that P(s) = t and P(t) = s. Prove that b - st is a root of the equation x^{2} =ax = b - st = 0.

Words such as question are uninformative in titles. Please edit the title so that it better describes the specifics of your question. Do not hesitate to make it longer or include a formula if needed. More tips here. (autocomment)Normal Human 21 secs ago
0
Q: Why do we may suppose $a=0$ in this proof?

user42912I didn't understand why we may suppose $a=0$ in this proof: I'm reading Conway's Complex Analysis book, page 31. Any help is welcome

0
Q: Inequality of two numbers

user256670The question is "For two consecutive multiples of 5, the smaller number is greater than half of the larger one. Find the least values of these two number." I am stuck!

Short title. Short question. Inequality of two numbers
0
Q: Groups with center {e}

Peter For which numbers $n$ exists a group of order $n$ with center {e} ? And how many groups are there for given order ? The first such numbers are $6,10,12,14,18,20,21,...$ Groups with order $32,40,64$, for example have not center {e}. For $n=18$, we have $2$ groups with center {e} Is there a...

0
Q: can you do it?#challenge

shaik.saleemIF a set A has K elements,formulate a conjecture about the number of elements in p(A). I can't even understand the term conjecture.

Short title. Short question. can you do it?#challenge
0
Q: Line Integrals - Calculus

the manI have a problem asking me to find $\int_C \textbf{f} \cdot d\textbf{r}$ where $\textbf{f}$ = $(siny,xcosy)$, and the curve $C$ is any closed circle. I'm struggling with this, so far I have found that $d\textbf{r}$ is $(-rsin\theta,rcos\theta)d\theta$, which is then $(-y,x)d\theta$ in cartesian ...

0
Q: Proving S = {Ax | x in R^n} is closed under addition.

user297590To show that S is closed under addition, is it enough to say the following? Let u and v be in R^n. Then u + v is in R^n. Therefore A(u) + A(v) = A(u+v) and S is closed under addition. Thank you in advance!

Welcome to Math.SE, user297590. This site uses MathJax formatting of formulas. More tips here. (autocomment)Normal Human 20 secs ago
0
Q: Inequality on integer n>1

Math-BoyIf $ n $ be a positive integer $>1$, prove that $$2^{n(n+1)}\gt(n+1)^{n+1}\biggl(\frac{n}{1}\biggr)^{n}\biggl(\frac{n-1}{2}\biggr)^{n-1}...\biggl(\frac{2}{n-1}\biggr)^{2}\biggl(\frac{1}{n}\biggr)$$

Short title. Title ends with a digit. Inequality on integer n>1
0
Q: Stuck on the last two parts of this 4-part problem dealing with Bilinear Forms,

User001Let A and B be two real $nxn$ matrices. Part 1) Show that if A and B are symmetric, then for any $\vec x \in R^n$, $$((A^2 + B^2)\vec x, \vec x) \ge((AB+BA)\vec x, \vec x)$$ Hint: look at $((A−B)^2 \vec x,\vec x)$ Part 2) Find a counterexample to Part (1) if the assumption of symmetry is d...

0
Q: Zero-infinity hypothesis

Federico Pohlsmath.stackexchange community. I have joined to inquire on a hypothesis a friend of mine has recently proposed. Please note: before posting this, I have repetitively told him that his logic is flawed in a plethora of ways. But I'll humor him, just this once. This dear friend of mine claims that w...

0
Q: Relation between Moment Generating Function of different algebraic equations of identically distributed random variables

PSKLets say we have a series random variables $h_{i,j}$ Now if $\gamma(i) = \sum^{J}_{j=1} {h_{i,j}}^2$ has a an MGF $M_{\gamma(i)}(s)$ and $\beta(i,\tilde{i}) = \sum^{J}_{j = 1} |{h_{\tilde{i},j}-h_{i,j}}|^2$ has an MGF $M_{\beta(i,\tilde{i})}(s)$ if the random variables ($h_{i,j}$) are identical...

Welcome to Math.SE, PSK. Consider adding a tag for a broader subject area to which the question belongs. Some of these tags might fit. (from a bot)Normal Human 21 secs ago
0
Q: Does any continuous function from the open unit interval $(0,1)$ to itself has a fixed point?

Vikrant DesaiI know that the result is true for closed interval $[0,1]$ by using intermediate value property. But in the case where we consider open interval $(0,1)$ does the solution change?

0
Q: Find Normal from Reflection

Cos GlynosI have a vector A with direction $\vec D$. I also have another vector B with direction $\vec R$. (think of $\vec R$ to be the reflection of $\vec D$ at point B.) From the reflection formula:$$ \vec R = \vec D-2*(\vec D \cdot \vec N)*\vec N $$ (we know both $\vec R$ and $\vec D$) How can I fin...

0
Q: Intuition behind rejection sampling proof question

guestI have a quick question about the proof of rejection sampling. Suppose we know how to sample from a distribution with $Y$ pdf $q$, and want to sample from a distribution $X$ with (known) pdf $\pi$. Suppose we also know that $\pi(x) \leq M q(x)$ for some constant $M>0$ and for all $x$. Then rejec...

Words such as question do not add information to titles. Please edit the title so that it better describes the specifics of your question. Do not hesitate to make it longer or include a formula if needed. More tips here. (from a bot)Normal Human 21 secs ago
 
2:04 PM
4
Q: What is the correct procedure for requesting that two questions be merged?

MrsEdThere was some discussion in SOCVR chat whether these two questions should be merged. What is the difference between parseInt() and Number()? What is the difference between parseInt(string) and Number(string) in JavaScript? There was a lack of clarity — do we need to post the request on Meta,...

0
Q: Is it possible to receive multiple bounties for same question?

Horst JahnsI have recently replied to a question on stackoverflow with a bounty of 50 reputation. If you do not award your bounty within 7 days (plus the grace period), the highest voted answer created after the bounty started with a minimum score of 2 will be awarded half the bounty amount. This...

 
0
Q: Jigsaw Equation

Brendan AnnableI'm looking for an equation that looks like the image below. I am comfortable constructing functions that pass the vertical line test, but as this is a relation, I am not really sure how to start. Would this need to be defined implicitly? An explanation on how you constructed it would be great...

Short title. Jigsaw Equation
0
Q: Question on countably compact, sequentially compact and compact

Henry WenIs there a countably compact space which is neither sequentially compact nor compact? Many thanks!

0
Q: Kernal in Modern Algbra

Ernest Michael NelsonWhat is a Kernel and how can it be describe in the real world and how can it be defined well and precisely. Tried asking my professor and he just tell us it is just abstract idea.

Short title. Short question. Kernal in Modern Algbra
 
0
Q: Providing Code Discription Answers

Jean-Pierre OosthuizenA lot of answers have no 'comments' in the code, specifically answer which have changed the OP script or developed something different. Example of providing comments in your Answer: 'For each visible cell in Column 2 "Comment" For Each cell In ContractRng.SpecialCells(xlCellTypeVisible)...

 
2:27 PM
0
Q: Evaluate Area of a surface

JAEMTOThe cylinder $x^2$ + $y^2$ = $x$ divdies the unit sphere S into two regions $S_1$ and $S_2$, where $S_1$ is inside the cylinder and $S_2$ outside. Find the ratio of areas $A(S_2)$/$A(S_1)$ I think the area is not surface of a revolution, right? So, I think I should parametrize it, but I can't. ...

0
Q: Find a standard basis vector for r3

Mikael SierraFind a standard basis vector for r3 that can be added to the set [v1, v2] to produce a basis for r3 v1=(1,1,1), v2=(2,-1,3) So, from what i know, a standard basis vector in r3 is either v1=(1,0,0), v2=(0,1,0) or v3=(0,0,1). I have no clue but I would like to assume that I just add the standard ...

Title ends with a digit. Find a standard basis vector for r3
0
Q: Prove that $\cos\left(\frac{2\pi}{n}\right)+\cos\left(\frac{4\pi}{n}\right)+\ldots+\cos\left(\frac{2(n-1)\pi}{n}\right)=-1$

nullgeppettoMay you help on how to start, or where to look for the following question? By using the $n$-th roots of the unity, show that: $\cos\left(\frac{2\pi}{n}\right)+\cos\left(\frac{4\pi}{n}\right)+\ldots+\cos\left(\frac{2(n-1)\pi}{n}\right)=-1$ $\sin\left(\frac{2\pi}{n}\right)+\sin\left(\frac{4\pi}{...

0
Q: Question about the det. of the Schur complement

tamSuppose $Z$ is a complex (Wishart) matrix. Let $a=\frac{1}{[Z^{-1}]_{kk}}$, where $Z^{-1}$ is the inverse of $Z$ and $[Z^{-1}]_{kk}$ represents the $(k,k)$-th entry of $Z^{-1}$. When I was reading an article, I saw that they define $Z_{kk}^{\text{SC}}$ as the Schur complement of $[Z]_{kk}$, a...

Words such as question do not add information to titles. Please edit the title so that it better describes the specifics of your question. Do not hesitate to make it longer or include a formula if needed. More tips here. (autocomment)Normal Human 21 secs ago
 
0
Q: Is it allowed to ask a question about specific university and department on stackexchange?

SSimonDear members and moderators, are you allowing questions in this forum about specific PhD programs and scholarship offers? Is it appropriate and ethical to raise a question about some specific department and research members of a PhD program, anecdotal experiences of members on that department, in...

 
0
Q: Permutation/Combination problem - choosing 2 objects from different categories

user297605Context: there are three candies in a basket (5 red, 4 blue, 5 green). How many ways can you choose 2 candies with different color? Should I use permutation or combination w/ this? I tried using: 14!/(14-2)! = 182 ways; but it's marked as wrong.

0
Q: complex integral with residue theorem

Olya12$\oint\limits_{|z-3|=4}^{}\frac{z}{cosz-1}dz$ I think $cosz=1$ $z=2\pi k$ to include only $z=0$ and $z=2\pi$ But how next?

 
2:47 PM
0
Q: Need a relation that is only reflexive

Jonhy HubeettoNeed a relation between a and b that is only reflexive. Example: a < b is only transitive. Thanks

0
Q: Additional condition to make a difficult inequality true

QuantumLogarithmConsider the quantity, $$\frac{\sum\limits_{i=0}^{n} A_i}{\sum\limits_{i=0}^{n} B_i},$$ where $A_i \leq B_i$ for any $i$ and $A_i$ is increasing with $i$. Let $S_i$ be exponentially decreasing with $i$. Let all these numbers be positive strictly less than $1$. Which additional conditions do I ne...

Words such as difficult are uninformative in titles. Please edit the title so that it better describes the specifics of your question. Do not hesitate to make it longer or include a formula if needed. More tips here. (autocomment)Normal Human 21 secs ago
 
00:00 - 15:0015:00 - 00:00

« first day (38 days earlier)      last day (536 days later) »